What actually is the vector of angular momentum?Angular momentum about a moving axisAngular momentum and RotationAngular Momentum vs Moment of InertiaMoment of inertia of a football and its angular momentumCollision of Discs and Snooker KicksDeviation of free falling objects (Coriolis effect) using conservation of angular momentumGyroscopic Force/Torque of a Constrained WheelCan the direction of angular momentum and angular velocity differ?How can angular momentum not be parallel with angular velocity?Question about Angular MomentumWhere does the energy used to rotate the axis of a flywheel orthogonally go?The moment of inertia of a spinning gyroscope assemblyCan conservation of angular momentum be derived from energy consideration?

Was the Highlands Ranch shooting the 115th mass shooting in the US in 2019

Series that evaluates to different values upon changing order of summation

Ex-manager wants to stay in touch, I don't want to

How does the Time Stop spell work when being grappled?

Is it a good idea to copy a trader when investing?

Pre-1993 comic in which Wolverine's claws were turned to rubber?

How to get a ellipse shaped node in Tikz Network?

Is there a need for better software for writers?

Watching the game, having a puzzle

Windows OS quantum vs. SQL OS Quantum

Best species to breed to intelligence

No such column 'DeveloperName' on entity 'RecordType' after Summer '19 release on sandbox

Was there a contingency plan in place if Little Boy failed to detonate?

How to handle DM constantly stealing everything from sleeping characters?

Improving Sati-Sampajañña (situative wisdom)

What is the name of meteoroids which hit Moon, Mars, or pretty much anything that isn’t the Earth?

Names of the Six Tastes

Exception propagation: When to catch exceptions?

What can cause an unfrozen indoor copper drain pipe to crack?

Why are parallelograms defined as quadrilaterals? What term would encompass polygons with greater than two parallel pairs?

How to evaluate sum with one million summands?

Why is PerfectForwardSecrecy considered OK, when it has same defects as salt-less password hashing?

When quoting someone, is it proper to change "gotta" to "got to" without modifying the rest of the quote?

We are two immediate neighbors who forged our own powers to form concatenated relationship. Who are we?



What actually is the vector of angular momentum?


Angular momentum about a moving axisAngular momentum and RotationAngular Momentum vs Moment of InertiaMoment of inertia of a football and its angular momentumCollision of Discs and Snooker KicksDeviation of free falling objects (Coriolis effect) using conservation of angular momentumGyroscopic Force/Torque of a Constrained WheelCan the direction of angular momentum and angular velocity differ?How can angular momentum not be parallel with angular velocity?Question about Angular MomentumWhere does the energy used to rotate the axis of a flywheel orthogonally go?The moment of inertia of a spinning gyroscope assemblyCan conservation of angular momentum be derived from energy consideration?













0












$begingroup$


If an object spins around a central point, it gets angular momentum which is a vector with an orientation dependent on whether its clockwise rotation or anticlockwise, i get that. But what the vector part actually is, is confusing me. Does it mean that the object spinning gets a force going upwards or downwards? In that case, how do you explain bicycles? Shouldn't the spinning result in angular momentum pointing in one direction thus forcing your bike to fall over the same way someone would if they pushed you from either side of the bike?










share|cite|improve this question











$endgroup$







  • 1




    $begingroup$
    Your question seems to be not clear. Try to improve your question
    $endgroup$
    – Tojrah
    Apr 30 at 13:54






  • 1




    $begingroup$
    The angular momentum is defined as $mathbfL=mathbfrtimesmathbfp$. If you calculate its derivative with respect to time you will see it can be expressed in terms of angular velocity. No more than that, don't overengineer it, don't try to "interpret" it.
    $endgroup$
    – gented
    Apr 30 at 14:02






  • 1




    $begingroup$
    Why would there be a force upwards just because angular momentum points up?
    $endgroup$
    – jacob1729
    Apr 30 at 14:55






  • 8




    $begingroup$
    Angular momentum isn't really a vector, it's a pseudo vector (aka axial vector) which is the antisymmetric part of a rank-2 tensor, so thinking of it like a (vector) force will most likely lead to confusion. They are different geometrical objects.
    $endgroup$
    – JEB
    Apr 30 at 15:39






  • 1




    $begingroup$
    @JEB - angular momentum and toque are vectors, but they are free vectors as opposed to linear momentum and force which are line vectors. Line vectors operating along a line in space and free vectors don't have a location, just a direction, and magnitude.
    $endgroup$
    – ja72
    Apr 30 at 18:41















0












$begingroup$


If an object spins around a central point, it gets angular momentum which is a vector with an orientation dependent on whether its clockwise rotation or anticlockwise, i get that. But what the vector part actually is, is confusing me. Does it mean that the object spinning gets a force going upwards or downwards? In that case, how do you explain bicycles? Shouldn't the spinning result in angular momentum pointing in one direction thus forcing your bike to fall over the same way someone would if they pushed you from either side of the bike?










share|cite|improve this question











$endgroup$







  • 1




    $begingroup$
    Your question seems to be not clear. Try to improve your question
    $endgroup$
    – Tojrah
    Apr 30 at 13:54






  • 1




    $begingroup$
    The angular momentum is defined as $mathbfL=mathbfrtimesmathbfp$. If you calculate its derivative with respect to time you will see it can be expressed in terms of angular velocity. No more than that, don't overengineer it, don't try to "interpret" it.
    $endgroup$
    – gented
    Apr 30 at 14:02






  • 1




    $begingroup$
    Why would there be a force upwards just because angular momentum points up?
    $endgroup$
    – jacob1729
    Apr 30 at 14:55






  • 8




    $begingroup$
    Angular momentum isn't really a vector, it's a pseudo vector (aka axial vector) which is the antisymmetric part of a rank-2 tensor, so thinking of it like a (vector) force will most likely lead to confusion. They are different geometrical objects.
    $endgroup$
    – JEB
    Apr 30 at 15:39






  • 1




    $begingroup$
    @JEB - angular momentum and toque are vectors, but they are free vectors as opposed to linear momentum and force which are line vectors. Line vectors operating along a line in space and free vectors don't have a location, just a direction, and magnitude.
    $endgroup$
    – ja72
    Apr 30 at 18:41













0












0








0


2



$begingroup$


If an object spins around a central point, it gets angular momentum which is a vector with an orientation dependent on whether its clockwise rotation or anticlockwise, i get that. But what the vector part actually is, is confusing me. Does it mean that the object spinning gets a force going upwards or downwards? In that case, how do you explain bicycles? Shouldn't the spinning result in angular momentum pointing in one direction thus forcing your bike to fall over the same way someone would if they pushed you from either side of the bike?










share|cite|improve this question











$endgroup$




If an object spins around a central point, it gets angular momentum which is a vector with an orientation dependent on whether its clockwise rotation or anticlockwise, i get that. But what the vector part actually is, is confusing me. Does it mean that the object spinning gets a force going upwards or downwards? In that case, how do you explain bicycles? Shouldn't the spinning result in angular momentum pointing in one direction thus forcing your bike to fall over the same way someone would if they pushed you from either side of the bike?







angular-momentum vectors rotational-kinematics moment-of-inertia angular-velocity






share|cite|improve this question















share|cite|improve this question













share|cite|improve this question




share|cite|improve this question








edited Apr 30 at 18:38









Qmechanic

109k122041258




109k122041258










asked Apr 30 at 13:41









Daniel .SDaniel .S

71




71







  • 1




    $begingroup$
    Your question seems to be not clear. Try to improve your question
    $endgroup$
    – Tojrah
    Apr 30 at 13:54






  • 1




    $begingroup$
    The angular momentum is defined as $mathbfL=mathbfrtimesmathbfp$. If you calculate its derivative with respect to time you will see it can be expressed in terms of angular velocity. No more than that, don't overengineer it, don't try to "interpret" it.
    $endgroup$
    – gented
    Apr 30 at 14:02






  • 1




    $begingroup$
    Why would there be a force upwards just because angular momentum points up?
    $endgroup$
    – jacob1729
    Apr 30 at 14:55






  • 8




    $begingroup$
    Angular momentum isn't really a vector, it's a pseudo vector (aka axial vector) which is the antisymmetric part of a rank-2 tensor, so thinking of it like a (vector) force will most likely lead to confusion. They are different geometrical objects.
    $endgroup$
    – JEB
    Apr 30 at 15:39






  • 1




    $begingroup$
    @JEB - angular momentum and toque are vectors, but they are free vectors as opposed to linear momentum and force which are line vectors. Line vectors operating along a line in space and free vectors don't have a location, just a direction, and magnitude.
    $endgroup$
    – ja72
    Apr 30 at 18:41












  • 1




    $begingroup$
    Your question seems to be not clear. Try to improve your question
    $endgroup$
    – Tojrah
    Apr 30 at 13:54






  • 1




    $begingroup$
    The angular momentum is defined as $mathbfL=mathbfrtimesmathbfp$. If you calculate its derivative with respect to time you will see it can be expressed in terms of angular velocity. No more than that, don't overengineer it, don't try to "interpret" it.
    $endgroup$
    – gented
    Apr 30 at 14:02






  • 1




    $begingroup$
    Why would there be a force upwards just because angular momentum points up?
    $endgroup$
    – jacob1729
    Apr 30 at 14:55






  • 8




    $begingroup$
    Angular momentum isn't really a vector, it's a pseudo vector (aka axial vector) which is the antisymmetric part of a rank-2 tensor, so thinking of it like a (vector) force will most likely lead to confusion. They are different geometrical objects.
    $endgroup$
    – JEB
    Apr 30 at 15:39






  • 1




    $begingroup$
    @JEB - angular momentum and toque are vectors, but they are free vectors as opposed to linear momentum and force which are line vectors. Line vectors operating along a line in space and free vectors don't have a location, just a direction, and magnitude.
    $endgroup$
    – ja72
    Apr 30 at 18:41







1




1




$begingroup$
Your question seems to be not clear. Try to improve your question
$endgroup$
– Tojrah
Apr 30 at 13:54




$begingroup$
Your question seems to be not clear. Try to improve your question
$endgroup$
– Tojrah
Apr 30 at 13:54




1




1




$begingroup$
The angular momentum is defined as $mathbfL=mathbfrtimesmathbfp$. If you calculate its derivative with respect to time you will see it can be expressed in terms of angular velocity. No more than that, don't overengineer it, don't try to "interpret" it.
$endgroup$
– gented
Apr 30 at 14:02




$begingroup$
The angular momentum is defined as $mathbfL=mathbfrtimesmathbfp$. If you calculate its derivative with respect to time you will see it can be expressed in terms of angular velocity. No more than that, don't overengineer it, don't try to "interpret" it.
$endgroup$
– gented
Apr 30 at 14:02




1




1




$begingroup$
Why would there be a force upwards just because angular momentum points up?
$endgroup$
– jacob1729
Apr 30 at 14:55




$begingroup$
Why would there be a force upwards just because angular momentum points up?
$endgroup$
– jacob1729
Apr 30 at 14:55




8




8




$begingroup$
Angular momentum isn't really a vector, it's a pseudo vector (aka axial vector) which is the antisymmetric part of a rank-2 tensor, so thinking of it like a (vector) force will most likely lead to confusion. They are different geometrical objects.
$endgroup$
– JEB
Apr 30 at 15:39




$begingroup$
Angular momentum isn't really a vector, it's a pseudo vector (aka axial vector) which is the antisymmetric part of a rank-2 tensor, so thinking of it like a (vector) force will most likely lead to confusion. They are different geometrical objects.
$endgroup$
– JEB
Apr 30 at 15:39




1




1




$begingroup$
@JEB - angular momentum and toque are vectors, but they are free vectors as opposed to linear momentum and force which are line vectors. Line vectors operating along a line in space and free vectors don't have a location, just a direction, and magnitude.
$endgroup$
– ja72
Apr 30 at 18:41




$begingroup$
@JEB - angular momentum and toque are vectors, but they are free vectors as opposed to linear momentum and force which are line vectors. Line vectors operating along a line in space and free vectors don't have a location, just a direction, and magnitude.
$endgroup$
– ja72
Apr 30 at 18:41










8 Answers
8






active

oldest

votes


















9












$begingroup$

Angular momentum has a magnitude, and is about some axis (thus has some sense of "direction). So vectors are used to represent the quantity, and much of the machinery of vectors applies to it. However, there are ways in which "vector" is a misleading term when applied to it, and some people characterize them as being in other mathematical structures, such as a Clifford algebra. Regardless of the exact mathematical formulation, they do not live in the same "space" as the physical world. The vector that represents angular momentum has physical meaning, but it doesn't physically exist. If an angular momentum is represented by a vector pointing east, that doesn't mean that there is any physical thing pointing east. It just means that the angular momentum is around an east-west axis, and that it is "positive" in the eastern direction, where "positive" is an arbitrary convention that we use to keep track of direction.



And apart from all the above, angular momentum is not a force, any more than linear momentum is. It is rotation around an axis.






share|cite|improve this answer









$endgroup$




















    4












    $begingroup$

    1. Angular momentum of a particle only describes the location of the linear momentum vector $boldsymbolp = m boldsymbolv$ where $boldsymbolv$ is the linear velocity and $m$ the mass. Momentum acts along a line in space called the axis of percussion.


    2. Angular momentum $boldsymbolL = boldsymbolr times boldsymbolp$ as measured at the origin, when a particle is located at $boldsymbolr$. It measures the moment of momentum in that the location of momentum can be recovered by $$boldsymbolr = frac boldsymbolp times boldsymbolL boldsymbolp $$


    3. This is entirely analogous to torque, which is the moment of force. The torque vector tells us the location of the line of action of a force with $$ boldsymbolr = frac boldsymbolF times boldsymboltau boldsymbolF $$


    4. It is also entirely analogous to linear velocity being the moment of rotation. The linear velocity of a rotating rigid body measured at the origin is $boldsymbolv = boldsymbolr times boldsymbolomega$ were $boldsymbolr$ is the location of the rotation axis.


    5. The location of the rotation axis is recovered similarly with $$ boldsymbolr = frac boldsymbolomega times boldsymbolv boldsymbolomega $$


    6. For rigid bodies, angular momentum can exist without linear momentum when a body is rotating about its center of mass. The interpretation here is the there are particles that are rotating about an axis and the direction of angular momentum vector is somewhat related (but not exactly) to the direction of rotation $$boldsymbolL = mathrmI boldsymbolomega$$ It represents the direction you need to apply a torque at in order to instantaneously stop a rotating body.



    7. The full expression of angular momentum of a rigid body includes the motion of the center of mass, as well as the motion about the center of mass



      $$ beginaligned
      boldsymbolp & = m , boldsymbolv_rm com \
      boldsymbolL & = mathrmI_rm com boldsymbolomega + boldsymbolr_rm com times boldsymbolp endaligned $$



    See this similar post as well as this more extensive post for more details.






    share|cite|improve this answer











    $endgroup$








    • 1




      $begingroup$
      It looks like you have a typo in 1. Angular momentum of a particle only describes the location of the linear momentum vector p=mv
      $endgroup$
      – PM 2Ring
      Apr 30 at 20:16










    • $begingroup$
      @PM2Ring - I still don't see it. Which word has the typo? Thanks.
      $endgroup$
      – ja72
      May 1 at 11:59






    • 1




      $begingroup$
      Ok, maybe I just don't get what you're trying to say. How does the angular momentum of a particle describe the location of the linear momentum vector?
      $endgroup$
      – PM 2Ring
      May 1 at 12:02










    • $begingroup$
      @PM2Ring Use the first equation above to get the location of momentum. Angular momentum is just the moment of linear momentum, just as torque is the moment of force. This is just saying that angular momentum describes linear momentum at a distance, just as toque describes a force at a distance. The term moment of mathematically means $$rm quantity = rm position times rm something$$
      $endgroup$
      – ja72
      May 1 at 18:27



















    3












    $begingroup$

    • The vector form of a linear property (force, velocity, momentum...) is a mathematical tool telling direction.


    • The vector form of a rotational property (torque, angular velocity, angular momentum...) is a mathematical tool telling rotational direction.


    By rotational direction we just mean the right-hand-rule, so, a way to determine whether the rotation is clockwise or counterclockwise.



    Don't mix the two. Vector forms are mathematical inventions that can represent physical properties depending on the situation. They mean different things for different types of properties. They sometimes feel fairly "intuitive" (when they define direction) whereas they other times do not make much physical sense but are "pseudo" vectors (when they define an imaginary axis about which we rotate).






    share|cite|improve this answer











    $endgroup$




















      1












      $begingroup$

      The angular momentum vector has a magnitude and a direction, just like any other vector. However, there is an issue with angular momentum that doesn't apply for objects moving in a linear direction.



      If you apply torque to the outside rim of a bicycle wheel, it spins in the direction of the applied force that is producing that torque. That bicycle wheel acquires angular momentum as a result. If you look at one side of the bicycle wheel, you would say that it's spinning in the clockwise direction. Somebody on the other side of the bicycle wheel would say that it's spinning in the counter-clockwise direction. Thus, the two of you will never agree because you are in different reference frames. This means that something must be defined for the angular momentum vector that doesn't change depending on the reference frame.



      By convention, if you use your right hand, and let your fingers move in the direction of the rotating bicycle wheel as you close your hand, with your thumb pointing "up" relative to your hand, your thumb will point in a direction that is perpendicular to the plane of rotation of the bicycle wheel and it will also point in the direction of the angular momentum vector. A person on the other side of the rotating bicycle wheel, performing the same right-hand rule, would see his thumb pointing in the same direction as your thumb, meaning that you would both now agree on the direction of the angular momentum vector.



      Regarding the bicycle not falling over as you are riding down the road, conservation of angular momentum means that the rotating bicycle wheels will "try" to keep their orientation in space as you ride down the road. If the wheels are spinning in a vertical direction, they will keep doing that until a torque is applied to them to change that orientation. Thus, angular momentum of the bicycle wheels stabilizes the bicycle, and the higher the speed of the wheels the more angular momentum that they have, which is why bicycles become easier to balance as the speed of rotation of the wheels increases.






      share|cite|improve this answer









      $endgroup$












      • $begingroup$
        Conservation of angular momentum has only a small influence on bicycle stability. See www2.eng.cam.ac.uk/~hemh1/gyrobike.htm
        $endgroup$
        – PM 2Ring
        Apr 30 at 21:05


















      1












      $begingroup$

      This is one of those times where simplifications can get in the way.



      In more formal contexts, angular momentum is not actually a vector. It's something called a bi-vector. It is the bivector constructed by taking the exerior product of the vector from the axis of rotation to the object and its momentum, $L=xland p$, rather than their cross product $xtimes p$.



      However, there's a catch. Exterior products take a while to learn, and the reasons why you would want to use the are rather nuanced. And, very conveniently, in 3 dimensions, for every bivector $xland p$ there is a corresponding pseudovector $x times p$ called an "axial vector" for which the equations are identical. You'll do the same operations to manipulate the numbers because the same $|x||p|sintheta$ term will show up in both sets of math. (this stops being a universal thing that can be done as we go into higher dimensions)



      So we teach momentum as a cross product. The price of this is that the meaning of the direction of that vector is kind of murky, as you noticed. It's a whole lot easier to teach, but we lost a little intuition along the way.



      Representation of bivector



      If you look at the way one thinks of a bivector and how we visualize them, the momentum bivector is all within the plane of rotation. You have the exterior product between the displacement between the axis of rotation (clearly in plane) and the momentum of the object (clearly in plane). One thinks entirely in the plane of rotation. The only reason the out of plane vectors ever appeared was because we used the cross product to create an axial vector to represent the deeper bi-vector. If you avoid that extra step, then we never see anything out of plane.



      So we teach angular momentum using axial vectors, because it makes it easier to teach. We don't need to teach new operators. However, at a deeper conceptual level, they are more correctly bivectors, and that should help explain why that non-intuitive out-of-plane angular momentum feels out of place... because it is. It's not wrong, just a little troublesome for intuition.






      share|cite|improve this answer









      $endgroup$




















        0












        $begingroup$


        What actually is the vector of angular momentum?




        It's the mathematical representation of physical rotation. That is, rotation can be quantified by a vector quantity. That's it. There's nothing "real" about the "vector of angular momentum." It's kind of like using complex numbers to represent wave phases. It's just a nice way to encode a real-world relationship into a mathematical model.



        The direction of the vector describes direction of planar rotation (right-hand rule) while the magnitude of the vector describes the magnitude of momentum. To speak of angular momentum, imagine two disks of equal size, one made of styrofoam, the other of granite. It will be much harder to accelerate the stone disk than the styrofoam disk. Similarly, once spinning it will be harder to decelerate the stone disk because it has higher angular momentum (due to more mass).






        share|cite|improve this answer









        $endgroup$




















          0












          $begingroup$

          You are encountering here an example of the subtle but significant difference between the physical quantity and its mathematical representation. The physical quantity is a measure of the degree to which mass is moving around some axis. There are many ways one could try to quantity that idea. The way that is adopted in physics is to define angular momentum so that it will be a conserved quantity in an isolated system. That is, while the internal parts of the system may move and push each in complicated ways, the total angular momentum of the system will not be affected by all such internal goings-on. This how we arrive at the definition
          $$
          bf L = bf r times bf v
          $$

          for the angular momentum of a particle about the origin. I am omitting the proof that this is indeed a nice conserved quantity, when added up over all the parts of an isolated system. For a rigidly rotating object, the resulting angular momentum vector is along the direction you know (i.e. along the axis of rotation, and pointing in the right-handed sense).



          What happens next is the mathematics says: "ok; you now have a useful quantity. And according to the way we defined it, it turns out to be a vector which points in the direction at right angles to both $bf r$ and $bf p$. If that direction seems puzzling to you, don't worry---it will become familiar as you see it working correctly in lots of examples. It does not mean anything is moving along the direction of $bf L$. Rather, the direction of $bf L$ indicates the direction of an axis about which some rotation or orbital motion is happening."



          That's the end of the answer to your question, but since you mention the example of a bicycle I'll say a little about that too.



          In the case of a bicycle the angular momentum associated with rotation of the wheels does not contribute much to the stability. What is much more important is the angle between the front fork and the axle, which determines how the weight of the bike tends to align the front wheel. However, just for interest, let's see what happens to the angular momentum of the wheels when you ride a bike.



          As you sit on the bike rolling forward, the angular momentum vector of either wheel is pointing to the left. (Use your right hand to check this). If we wish to turn the bike towards the right, we require, among other things, a torque to be applied to the front wheel so as to rotate its angular momentum vector. To make the correct change for a right turn the torque vector should be directed forwards, because we want the change $Delta bf L$ to be in the forwards direction, in order to swing $bf L$ around in the correct sense. One way to provide such a torque vector is to tip the wheel over, and the interesting point is that you should tip the wheel in the very sense that in any case you require in order to turn the bike, which is to say tipping into the corner, towards the right as you take a right hand bend (and towards the left as you take a left hand bend). (This tipping of the bike also ensures that the combination of friction and normal reaction from the road surface does not flip the bike over as you take the turn, and in fact this is its main role.)






          share|cite|improve this answer









          $endgroup$




















            0












            $begingroup$

            Whenever you see the cross product, know you’re dealing with a second rank antisymmetric tensor in disguise. Second rank tensors acting on 3D vectors can be represented by $3times3$ matrices, and if that matrix is required to be antisymmetric, that puts six constraints on it, leaving three degrees of freedom to define it. Thus the action of the antisymmetric tensor



            $$A=beginpmatrix
            0 & a_1 & a_2 \
            -a_1 & 0 & a_3 \
            -a_2 & -a_3 & 0
            endpmatrix$$



            on a vector $mathbfv,$ that is, $Amathbfv,$ can be conveniently denoted as $mathbfa times mathbfv,$ where $mathbfa = (-a_3, a_2, -a_1)^mathrmT.$ Let’s check:



            $$beginpmatrixa_x \ a_y \ a_zendpmatrix times
            beginpmatrixv_x \ v_y \ v_zendpmatrix =
            beginpmatrix
            0 & -a_z & a_y \
            a_z & 0 & -a_x \
            -a_y & a_x & 0
            endpmatrix
            beginpmatrixv_x \ v_y \ v_zendpmatrix =
            beginpmatrix
            -a_z v_y+a_yv_z \
            phantom-a_zv_x - a_xv_z \
            -a_yv_x+a_xv_y
            endpmatrix
            $$

            which matches the definition of the cross product in terms of the Cartesian coordinates.



            So it’s just a notational trick based on the fact that in 3D space (and in no other number of dimensions), rank 2 antisymmetric tensors have exactly as many degrees of freedom as vectors and can be put into correspondence. These pseudo-vectors end up pointing in directions which don’t necessarily mean anything is moving in that direction, and their magnitudes might be devoid of physical meaning as well.



            A similar approach can be used with other kinds of tensors, possibly requiring more than one vector to match the amount of information contained in the tensor. This is one way to define the $mathbfE$ and $mathbfB$ vectors in electromagnetism theory, while tensor forms of the laws combine those quantities in a single entity.






            share|cite|improve this answer









            $endgroup$













              Your Answer








              StackExchange.ready(function()
              var channelOptions =
              tags: "".split(" "),
              id: "151"
              ;
              initTagRenderer("".split(" "), "".split(" "), channelOptions);

              StackExchange.using("externalEditor", function()
              // Have to fire editor after snippets, if snippets enabled
              if (StackExchange.settings.snippets.snippetsEnabled)
              StackExchange.using("snippets", function()
              createEditor();
              );

              else
              createEditor();

              );

              function createEditor()
              StackExchange.prepareEditor(
              heartbeatType: 'answer',
              autoActivateHeartbeat: false,
              convertImagesToLinks: false,
              noModals: true,
              showLowRepImageUploadWarning: true,
              reputationToPostImages: null,
              bindNavPrevention: true,
              postfix: "",
              imageUploader:
              brandingHtml: "Powered by u003ca class="icon-imgur-white" href="https://imgur.com/"u003eu003c/au003e",
              contentPolicyHtml: "User contributions licensed under u003ca href="https://creativecommons.org/licenses/by-sa/3.0/"u003ecc by-sa 3.0 with attribution requiredu003c/au003e u003ca href="https://stackoverflow.com/legal/content-policy"u003e(content policy)u003c/au003e",
              allowUrls: true
              ,
              noCode: true, onDemand: true,
              discardSelector: ".discard-answer"
              ,immediatelyShowMarkdownHelp:true
              );



              );













              draft saved

              draft discarded


















              StackExchange.ready(
              function ()
              StackExchange.openid.initPostLogin('.new-post-login', 'https%3a%2f%2fphysics.stackexchange.com%2fquestions%2f476952%2fwhat-actually-is-the-vector-of-angular-momentum%23new-answer', 'question_page');

              );

              Post as a guest















              Required, but never shown

























              8 Answers
              8






              active

              oldest

              votes








              8 Answers
              8






              active

              oldest

              votes









              active

              oldest

              votes






              active

              oldest

              votes









              9












              $begingroup$

              Angular momentum has a magnitude, and is about some axis (thus has some sense of "direction). So vectors are used to represent the quantity, and much of the machinery of vectors applies to it. However, there are ways in which "vector" is a misleading term when applied to it, and some people characterize them as being in other mathematical structures, such as a Clifford algebra. Regardless of the exact mathematical formulation, they do not live in the same "space" as the physical world. The vector that represents angular momentum has physical meaning, but it doesn't physically exist. If an angular momentum is represented by a vector pointing east, that doesn't mean that there is any physical thing pointing east. It just means that the angular momentum is around an east-west axis, and that it is "positive" in the eastern direction, where "positive" is an arbitrary convention that we use to keep track of direction.



              And apart from all the above, angular momentum is not a force, any more than linear momentum is. It is rotation around an axis.






              share|cite|improve this answer









              $endgroup$

















                9












                $begingroup$

                Angular momentum has a magnitude, and is about some axis (thus has some sense of "direction). So vectors are used to represent the quantity, and much of the machinery of vectors applies to it. However, there are ways in which "vector" is a misleading term when applied to it, and some people characterize them as being in other mathematical structures, such as a Clifford algebra. Regardless of the exact mathematical formulation, they do not live in the same "space" as the physical world. The vector that represents angular momentum has physical meaning, but it doesn't physically exist. If an angular momentum is represented by a vector pointing east, that doesn't mean that there is any physical thing pointing east. It just means that the angular momentum is around an east-west axis, and that it is "positive" in the eastern direction, where "positive" is an arbitrary convention that we use to keep track of direction.



                And apart from all the above, angular momentum is not a force, any more than linear momentum is. It is rotation around an axis.






                share|cite|improve this answer









                $endgroup$















                  9












                  9








                  9





                  $begingroup$

                  Angular momentum has a magnitude, and is about some axis (thus has some sense of "direction). So vectors are used to represent the quantity, and much of the machinery of vectors applies to it. However, there are ways in which "vector" is a misleading term when applied to it, and some people characterize them as being in other mathematical structures, such as a Clifford algebra. Regardless of the exact mathematical formulation, they do not live in the same "space" as the physical world. The vector that represents angular momentum has physical meaning, but it doesn't physically exist. If an angular momentum is represented by a vector pointing east, that doesn't mean that there is any physical thing pointing east. It just means that the angular momentum is around an east-west axis, and that it is "positive" in the eastern direction, where "positive" is an arbitrary convention that we use to keep track of direction.



                  And apart from all the above, angular momentum is not a force, any more than linear momentum is. It is rotation around an axis.






                  share|cite|improve this answer









                  $endgroup$



                  Angular momentum has a magnitude, and is about some axis (thus has some sense of "direction). So vectors are used to represent the quantity, and much of the machinery of vectors applies to it. However, there are ways in which "vector" is a misleading term when applied to it, and some people characterize them as being in other mathematical structures, such as a Clifford algebra. Regardless of the exact mathematical formulation, they do not live in the same "space" as the physical world. The vector that represents angular momentum has physical meaning, but it doesn't physically exist. If an angular momentum is represented by a vector pointing east, that doesn't mean that there is any physical thing pointing east. It just means that the angular momentum is around an east-west axis, and that it is "positive" in the eastern direction, where "positive" is an arbitrary convention that we use to keep track of direction.



                  And apart from all the above, angular momentum is not a force, any more than linear momentum is. It is rotation around an axis.







                  share|cite|improve this answer












                  share|cite|improve this answer



                  share|cite|improve this answer










                  answered Apr 30 at 20:07









                  AcccumulationAcccumulation

                  3,194514




                  3,194514





















                      4












                      $begingroup$

                      1. Angular momentum of a particle only describes the location of the linear momentum vector $boldsymbolp = m boldsymbolv$ where $boldsymbolv$ is the linear velocity and $m$ the mass. Momentum acts along a line in space called the axis of percussion.


                      2. Angular momentum $boldsymbolL = boldsymbolr times boldsymbolp$ as measured at the origin, when a particle is located at $boldsymbolr$. It measures the moment of momentum in that the location of momentum can be recovered by $$boldsymbolr = frac boldsymbolp times boldsymbolL boldsymbolp $$


                      3. This is entirely analogous to torque, which is the moment of force. The torque vector tells us the location of the line of action of a force with $$ boldsymbolr = frac boldsymbolF times boldsymboltau boldsymbolF $$


                      4. It is also entirely analogous to linear velocity being the moment of rotation. The linear velocity of a rotating rigid body measured at the origin is $boldsymbolv = boldsymbolr times boldsymbolomega$ were $boldsymbolr$ is the location of the rotation axis.


                      5. The location of the rotation axis is recovered similarly with $$ boldsymbolr = frac boldsymbolomega times boldsymbolv boldsymbolomega $$


                      6. For rigid bodies, angular momentum can exist without linear momentum when a body is rotating about its center of mass. The interpretation here is the there are particles that are rotating about an axis and the direction of angular momentum vector is somewhat related (but not exactly) to the direction of rotation $$boldsymbolL = mathrmI boldsymbolomega$$ It represents the direction you need to apply a torque at in order to instantaneously stop a rotating body.



                      7. The full expression of angular momentum of a rigid body includes the motion of the center of mass, as well as the motion about the center of mass



                        $$ beginaligned
                        boldsymbolp & = m , boldsymbolv_rm com \
                        boldsymbolL & = mathrmI_rm com boldsymbolomega + boldsymbolr_rm com times boldsymbolp endaligned $$



                      See this similar post as well as this more extensive post for more details.






                      share|cite|improve this answer











                      $endgroup$








                      • 1




                        $begingroup$
                        It looks like you have a typo in 1. Angular momentum of a particle only describes the location of the linear momentum vector p=mv
                        $endgroup$
                        – PM 2Ring
                        Apr 30 at 20:16










                      • $begingroup$
                        @PM2Ring - I still don't see it. Which word has the typo? Thanks.
                        $endgroup$
                        – ja72
                        May 1 at 11:59






                      • 1




                        $begingroup$
                        Ok, maybe I just don't get what you're trying to say. How does the angular momentum of a particle describe the location of the linear momentum vector?
                        $endgroup$
                        – PM 2Ring
                        May 1 at 12:02










                      • $begingroup$
                        @PM2Ring Use the first equation above to get the location of momentum. Angular momentum is just the moment of linear momentum, just as torque is the moment of force. This is just saying that angular momentum describes linear momentum at a distance, just as toque describes a force at a distance. The term moment of mathematically means $$rm quantity = rm position times rm something$$
                        $endgroup$
                        – ja72
                        May 1 at 18:27
















                      4












                      $begingroup$

                      1. Angular momentum of a particle only describes the location of the linear momentum vector $boldsymbolp = m boldsymbolv$ where $boldsymbolv$ is the linear velocity and $m$ the mass. Momentum acts along a line in space called the axis of percussion.


                      2. Angular momentum $boldsymbolL = boldsymbolr times boldsymbolp$ as measured at the origin, when a particle is located at $boldsymbolr$. It measures the moment of momentum in that the location of momentum can be recovered by $$boldsymbolr = frac boldsymbolp times boldsymbolL boldsymbolp $$


                      3. This is entirely analogous to torque, which is the moment of force. The torque vector tells us the location of the line of action of a force with $$ boldsymbolr = frac boldsymbolF times boldsymboltau boldsymbolF $$


                      4. It is also entirely analogous to linear velocity being the moment of rotation. The linear velocity of a rotating rigid body measured at the origin is $boldsymbolv = boldsymbolr times boldsymbolomega$ were $boldsymbolr$ is the location of the rotation axis.


                      5. The location of the rotation axis is recovered similarly with $$ boldsymbolr = frac boldsymbolomega times boldsymbolv boldsymbolomega $$


                      6. For rigid bodies, angular momentum can exist without linear momentum when a body is rotating about its center of mass. The interpretation here is the there are particles that are rotating about an axis and the direction of angular momentum vector is somewhat related (but not exactly) to the direction of rotation $$boldsymbolL = mathrmI boldsymbolomega$$ It represents the direction you need to apply a torque at in order to instantaneously stop a rotating body.



                      7. The full expression of angular momentum of a rigid body includes the motion of the center of mass, as well as the motion about the center of mass



                        $$ beginaligned
                        boldsymbolp & = m , boldsymbolv_rm com \
                        boldsymbolL & = mathrmI_rm com boldsymbolomega + boldsymbolr_rm com times boldsymbolp endaligned $$



                      See this similar post as well as this more extensive post for more details.






                      share|cite|improve this answer











                      $endgroup$








                      • 1




                        $begingroup$
                        It looks like you have a typo in 1. Angular momentum of a particle only describes the location of the linear momentum vector p=mv
                        $endgroup$
                        – PM 2Ring
                        Apr 30 at 20:16










                      • $begingroup$
                        @PM2Ring - I still don't see it. Which word has the typo? Thanks.
                        $endgroup$
                        – ja72
                        May 1 at 11:59






                      • 1




                        $begingroup$
                        Ok, maybe I just don't get what you're trying to say. How does the angular momentum of a particle describe the location of the linear momentum vector?
                        $endgroup$
                        – PM 2Ring
                        May 1 at 12:02










                      • $begingroup$
                        @PM2Ring Use the first equation above to get the location of momentum. Angular momentum is just the moment of linear momentum, just as torque is the moment of force. This is just saying that angular momentum describes linear momentum at a distance, just as toque describes a force at a distance. The term moment of mathematically means $$rm quantity = rm position times rm something$$
                        $endgroup$
                        – ja72
                        May 1 at 18:27














                      4












                      4








                      4





                      $begingroup$

                      1. Angular momentum of a particle only describes the location of the linear momentum vector $boldsymbolp = m boldsymbolv$ where $boldsymbolv$ is the linear velocity and $m$ the mass. Momentum acts along a line in space called the axis of percussion.


                      2. Angular momentum $boldsymbolL = boldsymbolr times boldsymbolp$ as measured at the origin, when a particle is located at $boldsymbolr$. It measures the moment of momentum in that the location of momentum can be recovered by $$boldsymbolr = frac boldsymbolp times boldsymbolL boldsymbolp $$


                      3. This is entirely analogous to torque, which is the moment of force. The torque vector tells us the location of the line of action of a force with $$ boldsymbolr = frac boldsymbolF times boldsymboltau boldsymbolF $$


                      4. It is also entirely analogous to linear velocity being the moment of rotation. The linear velocity of a rotating rigid body measured at the origin is $boldsymbolv = boldsymbolr times boldsymbolomega$ were $boldsymbolr$ is the location of the rotation axis.


                      5. The location of the rotation axis is recovered similarly with $$ boldsymbolr = frac boldsymbolomega times boldsymbolv boldsymbolomega $$


                      6. For rigid bodies, angular momentum can exist without linear momentum when a body is rotating about its center of mass. The interpretation here is the there are particles that are rotating about an axis and the direction of angular momentum vector is somewhat related (but not exactly) to the direction of rotation $$boldsymbolL = mathrmI boldsymbolomega$$ It represents the direction you need to apply a torque at in order to instantaneously stop a rotating body.



                      7. The full expression of angular momentum of a rigid body includes the motion of the center of mass, as well as the motion about the center of mass



                        $$ beginaligned
                        boldsymbolp & = m , boldsymbolv_rm com \
                        boldsymbolL & = mathrmI_rm com boldsymbolomega + boldsymbolr_rm com times boldsymbolp endaligned $$



                      See this similar post as well as this more extensive post for more details.






                      share|cite|improve this answer











                      $endgroup$



                      1. Angular momentum of a particle only describes the location of the linear momentum vector $boldsymbolp = m boldsymbolv$ where $boldsymbolv$ is the linear velocity and $m$ the mass. Momentum acts along a line in space called the axis of percussion.


                      2. Angular momentum $boldsymbolL = boldsymbolr times boldsymbolp$ as measured at the origin, when a particle is located at $boldsymbolr$. It measures the moment of momentum in that the location of momentum can be recovered by $$boldsymbolr = frac boldsymbolp times boldsymbolL boldsymbolp $$


                      3. This is entirely analogous to torque, which is the moment of force. The torque vector tells us the location of the line of action of a force with $$ boldsymbolr = frac boldsymbolF times boldsymboltau boldsymbolF $$


                      4. It is also entirely analogous to linear velocity being the moment of rotation. The linear velocity of a rotating rigid body measured at the origin is $boldsymbolv = boldsymbolr times boldsymbolomega$ were $boldsymbolr$ is the location of the rotation axis.


                      5. The location of the rotation axis is recovered similarly with $$ boldsymbolr = frac boldsymbolomega times boldsymbolv boldsymbolomega $$


                      6. For rigid bodies, angular momentum can exist without linear momentum when a body is rotating about its center of mass. The interpretation here is the there are particles that are rotating about an axis and the direction of angular momentum vector is somewhat related (but not exactly) to the direction of rotation $$boldsymbolL = mathrmI boldsymbolomega$$ It represents the direction you need to apply a torque at in order to instantaneously stop a rotating body.



                      7. The full expression of angular momentum of a rigid body includes the motion of the center of mass, as well as the motion about the center of mass



                        $$ beginaligned
                        boldsymbolp & = m , boldsymbolv_rm com \
                        boldsymbolL & = mathrmI_rm com boldsymbolomega + boldsymbolr_rm com times boldsymbolp endaligned $$



                      See this similar post as well as this more extensive post for more details.







                      share|cite|improve this answer














                      share|cite|improve this answer



                      share|cite|improve this answer








                      edited May 1 at 12:00

























                      answered Apr 30 at 18:02









                      ja72ja72

                      21.2k433104




                      21.2k433104







                      • 1




                        $begingroup$
                        It looks like you have a typo in 1. Angular momentum of a particle only describes the location of the linear momentum vector p=mv
                        $endgroup$
                        – PM 2Ring
                        Apr 30 at 20:16










                      • $begingroup$
                        @PM2Ring - I still don't see it. Which word has the typo? Thanks.
                        $endgroup$
                        – ja72
                        May 1 at 11:59






                      • 1




                        $begingroup$
                        Ok, maybe I just don't get what you're trying to say. How does the angular momentum of a particle describe the location of the linear momentum vector?
                        $endgroup$
                        – PM 2Ring
                        May 1 at 12:02










                      • $begingroup$
                        @PM2Ring Use the first equation above to get the location of momentum. Angular momentum is just the moment of linear momentum, just as torque is the moment of force. This is just saying that angular momentum describes linear momentum at a distance, just as toque describes a force at a distance. The term moment of mathematically means $$rm quantity = rm position times rm something$$
                        $endgroup$
                        – ja72
                        May 1 at 18:27













                      • 1




                        $begingroup$
                        It looks like you have a typo in 1. Angular momentum of a particle only describes the location of the linear momentum vector p=mv
                        $endgroup$
                        – PM 2Ring
                        Apr 30 at 20:16










                      • $begingroup$
                        @PM2Ring - I still don't see it. Which word has the typo? Thanks.
                        $endgroup$
                        – ja72
                        May 1 at 11:59






                      • 1




                        $begingroup$
                        Ok, maybe I just don't get what you're trying to say. How does the angular momentum of a particle describe the location of the linear momentum vector?
                        $endgroup$
                        – PM 2Ring
                        May 1 at 12:02










                      • $begingroup$
                        @PM2Ring Use the first equation above to get the location of momentum. Angular momentum is just the moment of linear momentum, just as torque is the moment of force. This is just saying that angular momentum describes linear momentum at a distance, just as toque describes a force at a distance. The term moment of mathematically means $$rm quantity = rm position times rm something$$
                        $endgroup$
                        – ja72
                        May 1 at 18:27








                      1




                      1




                      $begingroup$
                      It looks like you have a typo in 1. Angular momentum of a particle only describes the location of the linear momentum vector p=mv
                      $endgroup$
                      – PM 2Ring
                      Apr 30 at 20:16




                      $begingroup$
                      It looks like you have a typo in 1. Angular momentum of a particle only describes the location of the linear momentum vector p=mv
                      $endgroup$
                      – PM 2Ring
                      Apr 30 at 20:16












                      $begingroup$
                      @PM2Ring - I still don't see it. Which word has the typo? Thanks.
                      $endgroup$
                      – ja72
                      May 1 at 11:59




                      $begingroup$
                      @PM2Ring - I still don't see it. Which word has the typo? Thanks.
                      $endgroup$
                      – ja72
                      May 1 at 11:59




                      1




                      1




                      $begingroup$
                      Ok, maybe I just don't get what you're trying to say. How does the angular momentum of a particle describe the location of the linear momentum vector?
                      $endgroup$
                      – PM 2Ring
                      May 1 at 12:02




                      $begingroup$
                      Ok, maybe I just don't get what you're trying to say. How does the angular momentum of a particle describe the location of the linear momentum vector?
                      $endgroup$
                      – PM 2Ring
                      May 1 at 12:02












                      $begingroup$
                      @PM2Ring Use the first equation above to get the location of momentum. Angular momentum is just the moment of linear momentum, just as torque is the moment of force. This is just saying that angular momentum describes linear momentum at a distance, just as toque describes a force at a distance. The term moment of mathematically means $$rm quantity = rm position times rm something$$
                      $endgroup$
                      – ja72
                      May 1 at 18:27





                      $begingroup$
                      @PM2Ring Use the first equation above to get the location of momentum. Angular momentum is just the moment of linear momentum, just as torque is the moment of force. This is just saying that angular momentum describes linear momentum at a distance, just as toque describes a force at a distance. The term moment of mathematically means $$rm quantity = rm position times rm something$$
                      $endgroup$
                      – ja72
                      May 1 at 18:27












                      3












                      $begingroup$

                      • The vector form of a linear property (force, velocity, momentum...) is a mathematical tool telling direction.


                      • The vector form of a rotational property (torque, angular velocity, angular momentum...) is a mathematical tool telling rotational direction.


                      By rotational direction we just mean the right-hand-rule, so, a way to determine whether the rotation is clockwise or counterclockwise.



                      Don't mix the two. Vector forms are mathematical inventions that can represent physical properties depending on the situation. They mean different things for different types of properties. They sometimes feel fairly "intuitive" (when they define direction) whereas they other times do not make much physical sense but are "pseudo" vectors (when they define an imaginary axis about which we rotate).






                      share|cite|improve this answer











                      $endgroup$

















                        3












                        $begingroup$

                        • The vector form of a linear property (force, velocity, momentum...) is a mathematical tool telling direction.


                        • The vector form of a rotational property (torque, angular velocity, angular momentum...) is a mathematical tool telling rotational direction.


                        By rotational direction we just mean the right-hand-rule, so, a way to determine whether the rotation is clockwise or counterclockwise.



                        Don't mix the two. Vector forms are mathematical inventions that can represent physical properties depending on the situation. They mean different things for different types of properties. They sometimes feel fairly "intuitive" (when they define direction) whereas they other times do not make much physical sense but are "pseudo" vectors (when they define an imaginary axis about which we rotate).






                        share|cite|improve this answer











                        $endgroup$















                          3












                          3








                          3





                          $begingroup$

                          • The vector form of a linear property (force, velocity, momentum...) is a mathematical tool telling direction.


                          • The vector form of a rotational property (torque, angular velocity, angular momentum...) is a mathematical tool telling rotational direction.


                          By rotational direction we just mean the right-hand-rule, so, a way to determine whether the rotation is clockwise or counterclockwise.



                          Don't mix the two. Vector forms are mathematical inventions that can represent physical properties depending on the situation. They mean different things for different types of properties. They sometimes feel fairly "intuitive" (when they define direction) whereas they other times do not make much physical sense but are "pseudo" vectors (when they define an imaginary axis about which we rotate).






                          share|cite|improve this answer











                          $endgroup$



                          • The vector form of a linear property (force, velocity, momentum...) is a mathematical tool telling direction.


                          • The vector form of a rotational property (torque, angular velocity, angular momentum...) is a mathematical tool telling rotational direction.


                          By rotational direction we just mean the right-hand-rule, so, a way to determine whether the rotation is clockwise or counterclockwise.



                          Don't mix the two. Vector forms are mathematical inventions that can represent physical properties depending on the situation. They mean different things for different types of properties. They sometimes feel fairly "intuitive" (when they define direction) whereas they other times do not make much physical sense but are "pseudo" vectors (when they define an imaginary axis about which we rotate).







                          share|cite|improve this answer














                          share|cite|improve this answer



                          share|cite|improve this answer








                          edited May 1 at 6:04

























                          answered Apr 30 at 18:54









                          SteevenSteeven

                          28.3k867115




                          28.3k867115





















                              1












                              $begingroup$

                              The angular momentum vector has a magnitude and a direction, just like any other vector. However, there is an issue with angular momentum that doesn't apply for objects moving in a linear direction.



                              If you apply torque to the outside rim of a bicycle wheel, it spins in the direction of the applied force that is producing that torque. That bicycle wheel acquires angular momentum as a result. If you look at one side of the bicycle wheel, you would say that it's spinning in the clockwise direction. Somebody on the other side of the bicycle wheel would say that it's spinning in the counter-clockwise direction. Thus, the two of you will never agree because you are in different reference frames. This means that something must be defined for the angular momentum vector that doesn't change depending on the reference frame.



                              By convention, if you use your right hand, and let your fingers move in the direction of the rotating bicycle wheel as you close your hand, with your thumb pointing "up" relative to your hand, your thumb will point in a direction that is perpendicular to the plane of rotation of the bicycle wheel and it will also point in the direction of the angular momentum vector. A person on the other side of the rotating bicycle wheel, performing the same right-hand rule, would see his thumb pointing in the same direction as your thumb, meaning that you would both now agree on the direction of the angular momentum vector.



                              Regarding the bicycle not falling over as you are riding down the road, conservation of angular momentum means that the rotating bicycle wheels will "try" to keep their orientation in space as you ride down the road. If the wheels are spinning in a vertical direction, they will keep doing that until a torque is applied to them to change that orientation. Thus, angular momentum of the bicycle wheels stabilizes the bicycle, and the higher the speed of the wheels the more angular momentum that they have, which is why bicycles become easier to balance as the speed of rotation of the wheels increases.






                              share|cite|improve this answer









                              $endgroup$












                              • $begingroup$
                                Conservation of angular momentum has only a small influence on bicycle stability. See www2.eng.cam.ac.uk/~hemh1/gyrobike.htm
                                $endgroup$
                                – PM 2Ring
                                Apr 30 at 21:05















                              1












                              $begingroup$

                              The angular momentum vector has a magnitude and a direction, just like any other vector. However, there is an issue with angular momentum that doesn't apply for objects moving in a linear direction.



                              If you apply torque to the outside rim of a bicycle wheel, it spins in the direction of the applied force that is producing that torque. That bicycle wheel acquires angular momentum as a result. If you look at one side of the bicycle wheel, you would say that it's spinning in the clockwise direction. Somebody on the other side of the bicycle wheel would say that it's spinning in the counter-clockwise direction. Thus, the two of you will never agree because you are in different reference frames. This means that something must be defined for the angular momentum vector that doesn't change depending on the reference frame.



                              By convention, if you use your right hand, and let your fingers move in the direction of the rotating bicycle wheel as you close your hand, with your thumb pointing "up" relative to your hand, your thumb will point in a direction that is perpendicular to the plane of rotation of the bicycle wheel and it will also point in the direction of the angular momentum vector. A person on the other side of the rotating bicycle wheel, performing the same right-hand rule, would see his thumb pointing in the same direction as your thumb, meaning that you would both now agree on the direction of the angular momentum vector.



                              Regarding the bicycle not falling over as you are riding down the road, conservation of angular momentum means that the rotating bicycle wheels will "try" to keep their orientation in space as you ride down the road. If the wheels are spinning in a vertical direction, they will keep doing that until a torque is applied to them to change that orientation. Thus, angular momentum of the bicycle wheels stabilizes the bicycle, and the higher the speed of the wheels the more angular momentum that they have, which is why bicycles become easier to balance as the speed of rotation of the wheels increases.






                              share|cite|improve this answer









                              $endgroup$












                              • $begingroup$
                                Conservation of angular momentum has only a small influence on bicycle stability. See www2.eng.cam.ac.uk/~hemh1/gyrobike.htm
                                $endgroup$
                                – PM 2Ring
                                Apr 30 at 21:05













                              1












                              1








                              1





                              $begingroup$

                              The angular momentum vector has a magnitude and a direction, just like any other vector. However, there is an issue with angular momentum that doesn't apply for objects moving in a linear direction.



                              If you apply torque to the outside rim of a bicycle wheel, it spins in the direction of the applied force that is producing that torque. That bicycle wheel acquires angular momentum as a result. If you look at one side of the bicycle wheel, you would say that it's spinning in the clockwise direction. Somebody on the other side of the bicycle wheel would say that it's spinning in the counter-clockwise direction. Thus, the two of you will never agree because you are in different reference frames. This means that something must be defined for the angular momentum vector that doesn't change depending on the reference frame.



                              By convention, if you use your right hand, and let your fingers move in the direction of the rotating bicycle wheel as you close your hand, with your thumb pointing "up" relative to your hand, your thumb will point in a direction that is perpendicular to the plane of rotation of the bicycle wheel and it will also point in the direction of the angular momentum vector. A person on the other side of the rotating bicycle wheel, performing the same right-hand rule, would see his thumb pointing in the same direction as your thumb, meaning that you would both now agree on the direction of the angular momentum vector.



                              Regarding the bicycle not falling over as you are riding down the road, conservation of angular momentum means that the rotating bicycle wheels will "try" to keep their orientation in space as you ride down the road. If the wheels are spinning in a vertical direction, they will keep doing that until a torque is applied to them to change that orientation. Thus, angular momentum of the bicycle wheels stabilizes the bicycle, and the higher the speed of the wheels the more angular momentum that they have, which is why bicycles become easier to balance as the speed of rotation of the wheels increases.






                              share|cite|improve this answer









                              $endgroup$



                              The angular momentum vector has a magnitude and a direction, just like any other vector. However, there is an issue with angular momentum that doesn't apply for objects moving in a linear direction.



                              If you apply torque to the outside rim of a bicycle wheel, it spins in the direction of the applied force that is producing that torque. That bicycle wheel acquires angular momentum as a result. If you look at one side of the bicycle wheel, you would say that it's spinning in the clockwise direction. Somebody on the other side of the bicycle wheel would say that it's spinning in the counter-clockwise direction. Thus, the two of you will never agree because you are in different reference frames. This means that something must be defined for the angular momentum vector that doesn't change depending on the reference frame.



                              By convention, if you use your right hand, and let your fingers move in the direction of the rotating bicycle wheel as you close your hand, with your thumb pointing "up" relative to your hand, your thumb will point in a direction that is perpendicular to the plane of rotation of the bicycle wheel and it will also point in the direction of the angular momentum vector. A person on the other side of the rotating bicycle wheel, performing the same right-hand rule, would see his thumb pointing in the same direction as your thumb, meaning that you would both now agree on the direction of the angular momentum vector.



                              Regarding the bicycle not falling over as you are riding down the road, conservation of angular momentum means that the rotating bicycle wheels will "try" to keep their orientation in space as you ride down the road. If the wheels are spinning in a vertical direction, they will keep doing that until a torque is applied to them to change that orientation. Thus, angular momentum of the bicycle wheels stabilizes the bicycle, and the higher the speed of the wheels the more angular momentum that they have, which is why bicycles become easier to balance as the speed of rotation of the wheels increases.







                              share|cite|improve this answer












                              share|cite|improve this answer



                              share|cite|improve this answer










                              answered Apr 30 at 20:49









                              David WhiteDavid White

                              5,0212621




                              5,0212621











                              • $begingroup$
                                Conservation of angular momentum has only a small influence on bicycle stability. See www2.eng.cam.ac.uk/~hemh1/gyrobike.htm
                                $endgroup$
                                – PM 2Ring
                                Apr 30 at 21:05
















                              • $begingroup$
                                Conservation of angular momentum has only a small influence on bicycle stability. See www2.eng.cam.ac.uk/~hemh1/gyrobike.htm
                                $endgroup$
                                – PM 2Ring
                                Apr 30 at 21:05















                              $begingroup$
                              Conservation of angular momentum has only a small influence on bicycle stability. See www2.eng.cam.ac.uk/~hemh1/gyrobike.htm
                              $endgroup$
                              – PM 2Ring
                              Apr 30 at 21:05




                              $begingroup$
                              Conservation of angular momentum has only a small influence on bicycle stability. See www2.eng.cam.ac.uk/~hemh1/gyrobike.htm
                              $endgroup$
                              – PM 2Ring
                              Apr 30 at 21:05











                              1












                              $begingroup$

                              This is one of those times where simplifications can get in the way.



                              In more formal contexts, angular momentum is not actually a vector. It's something called a bi-vector. It is the bivector constructed by taking the exerior product of the vector from the axis of rotation to the object and its momentum, $L=xland p$, rather than their cross product $xtimes p$.



                              However, there's a catch. Exterior products take a while to learn, and the reasons why you would want to use the are rather nuanced. And, very conveniently, in 3 dimensions, for every bivector $xland p$ there is a corresponding pseudovector $x times p$ called an "axial vector" for which the equations are identical. You'll do the same operations to manipulate the numbers because the same $|x||p|sintheta$ term will show up in both sets of math. (this stops being a universal thing that can be done as we go into higher dimensions)



                              So we teach momentum as a cross product. The price of this is that the meaning of the direction of that vector is kind of murky, as you noticed. It's a whole lot easier to teach, but we lost a little intuition along the way.



                              Representation of bivector



                              If you look at the way one thinks of a bivector and how we visualize them, the momentum bivector is all within the plane of rotation. You have the exterior product between the displacement between the axis of rotation (clearly in plane) and the momentum of the object (clearly in plane). One thinks entirely in the plane of rotation. The only reason the out of plane vectors ever appeared was because we used the cross product to create an axial vector to represent the deeper bi-vector. If you avoid that extra step, then we never see anything out of plane.



                              So we teach angular momentum using axial vectors, because it makes it easier to teach. We don't need to teach new operators. However, at a deeper conceptual level, they are more correctly bivectors, and that should help explain why that non-intuitive out-of-plane angular momentum feels out of place... because it is. It's not wrong, just a little troublesome for intuition.






                              share|cite|improve this answer









                              $endgroup$

















                                1












                                $begingroup$

                                This is one of those times where simplifications can get in the way.



                                In more formal contexts, angular momentum is not actually a vector. It's something called a bi-vector. It is the bivector constructed by taking the exerior product of the vector from the axis of rotation to the object and its momentum, $L=xland p$, rather than their cross product $xtimes p$.



                                However, there's a catch. Exterior products take a while to learn, and the reasons why you would want to use the are rather nuanced. And, very conveniently, in 3 dimensions, for every bivector $xland p$ there is a corresponding pseudovector $x times p$ called an "axial vector" for which the equations are identical. You'll do the same operations to manipulate the numbers because the same $|x||p|sintheta$ term will show up in both sets of math. (this stops being a universal thing that can be done as we go into higher dimensions)



                                So we teach momentum as a cross product. The price of this is that the meaning of the direction of that vector is kind of murky, as you noticed. It's a whole lot easier to teach, but we lost a little intuition along the way.



                                Representation of bivector



                                If you look at the way one thinks of a bivector and how we visualize them, the momentum bivector is all within the plane of rotation. You have the exterior product between the displacement between the axis of rotation (clearly in plane) and the momentum of the object (clearly in plane). One thinks entirely in the plane of rotation. The only reason the out of plane vectors ever appeared was because we used the cross product to create an axial vector to represent the deeper bi-vector. If you avoid that extra step, then we never see anything out of plane.



                                So we teach angular momentum using axial vectors, because it makes it easier to teach. We don't need to teach new operators. However, at a deeper conceptual level, they are more correctly bivectors, and that should help explain why that non-intuitive out-of-plane angular momentum feels out of place... because it is. It's not wrong, just a little troublesome for intuition.






                                share|cite|improve this answer









                                $endgroup$















                                  1












                                  1








                                  1





                                  $begingroup$

                                  This is one of those times where simplifications can get in the way.



                                  In more formal contexts, angular momentum is not actually a vector. It's something called a bi-vector. It is the bivector constructed by taking the exerior product of the vector from the axis of rotation to the object and its momentum, $L=xland p$, rather than their cross product $xtimes p$.



                                  However, there's a catch. Exterior products take a while to learn, and the reasons why you would want to use the are rather nuanced. And, very conveniently, in 3 dimensions, for every bivector $xland p$ there is a corresponding pseudovector $x times p$ called an "axial vector" for which the equations are identical. You'll do the same operations to manipulate the numbers because the same $|x||p|sintheta$ term will show up in both sets of math. (this stops being a universal thing that can be done as we go into higher dimensions)



                                  So we teach momentum as a cross product. The price of this is that the meaning of the direction of that vector is kind of murky, as you noticed. It's a whole lot easier to teach, but we lost a little intuition along the way.



                                  Representation of bivector



                                  If you look at the way one thinks of a bivector and how we visualize them, the momentum bivector is all within the plane of rotation. You have the exterior product between the displacement between the axis of rotation (clearly in plane) and the momentum of the object (clearly in plane). One thinks entirely in the plane of rotation. The only reason the out of plane vectors ever appeared was because we used the cross product to create an axial vector to represent the deeper bi-vector. If you avoid that extra step, then we never see anything out of plane.



                                  So we teach angular momentum using axial vectors, because it makes it easier to teach. We don't need to teach new operators. However, at a deeper conceptual level, they are more correctly bivectors, and that should help explain why that non-intuitive out-of-plane angular momentum feels out of place... because it is. It's not wrong, just a little troublesome for intuition.






                                  share|cite|improve this answer









                                  $endgroup$



                                  This is one of those times where simplifications can get in the way.



                                  In more formal contexts, angular momentum is not actually a vector. It's something called a bi-vector. It is the bivector constructed by taking the exerior product of the vector from the axis of rotation to the object and its momentum, $L=xland p$, rather than their cross product $xtimes p$.



                                  However, there's a catch. Exterior products take a while to learn, and the reasons why you would want to use the are rather nuanced. And, very conveniently, in 3 dimensions, for every bivector $xland p$ there is a corresponding pseudovector $x times p$ called an "axial vector" for which the equations are identical. You'll do the same operations to manipulate the numbers because the same $|x||p|sintheta$ term will show up in both sets of math. (this stops being a universal thing that can be done as we go into higher dimensions)



                                  So we teach momentum as a cross product. The price of this is that the meaning of the direction of that vector is kind of murky, as you noticed. It's a whole lot easier to teach, but we lost a little intuition along the way.



                                  Representation of bivector



                                  If you look at the way one thinks of a bivector and how we visualize them, the momentum bivector is all within the plane of rotation. You have the exterior product between the displacement between the axis of rotation (clearly in plane) and the momentum of the object (clearly in plane). One thinks entirely in the plane of rotation. The only reason the out of plane vectors ever appeared was because we used the cross product to create an axial vector to represent the deeper bi-vector. If you avoid that extra step, then we never see anything out of plane.



                                  So we teach angular momentum using axial vectors, because it makes it easier to teach. We don't need to teach new operators. However, at a deeper conceptual level, they are more correctly bivectors, and that should help explain why that non-intuitive out-of-plane angular momentum feels out of place... because it is. It's not wrong, just a little troublesome for intuition.







                                  share|cite|improve this answer












                                  share|cite|improve this answer



                                  share|cite|improve this answer










                                  answered May 1 at 1:10









                                  Cort AmmonCort Ammon

                                  25.3k45186




                                  25.3k45186





















                                      0












                                      $begingroup$


                                      What actually is the vector of angular momentum?




                                      It's the mathematical representation of physical rotation. That is, rotation can be quantified by a vector quantity. That's it. There's nothing "real" about the "vector of angular momentum." It's kind of like using complex numbers to represent wave phases. It's just a nice way to encode a real-world relationship into a mathematical model.



                                      The direction of the vector describes direction of planar rotation (right-hand rule) while the magnitude of the vector describes the magnitude of momentum. To speak of angular momentum, imagine two disks of equal size, one made of styrofoam, the other of granite. It will be much harder to accelerate the stone disk than the styrofoam disk. Similarly, once spinning it will be harder to decelerate the stone disk because it has higher angular momentum (due to more mass).






                                      share|cite|improve this answer









                                      $endgroup$

















                                        0












                                        $begingroup$


                                        What actually is the vector of angular momentum?




                                        It's the mathematical representation of physical rotation. That is, rotation can be quantified by a vector quantity. That's it. There's nothing "real" about the "vector of angular momentum." It's kind of like using complex numbers to represent wave phases. It's just a nice way to encode a real-world relationship into a mathematical model.



                                        The direction of the vector describes direction of planar rotation (right-hand rule) while the magnitude of the vector describes the magnitude of momentum. To speak of angular momentum, imagine two disks of equal size, one made of styrofoam, the other of granite. It will be much harder to accelerate the stone disk than the styrofoam disk. Similarly, once spinning it will be harder to decelerate the stone disk because it has higher angular momentum (due to more mass).






                                        share|cite|improve this answer









                                        $endgroup$















                                          0












                                          0








                                          0





                                          $begingroup$


                                          What actually is the vector of angular momentum?




                                          It's the mathematical representation of physical rotation. That is, rotation can be quantified by a vector quantity. That's it. There's nothing "real" about the "vector of angular momentum." It's kind of like using complex numbers to represent wave phases. It's just a nice way to encode a real-world relationship into a mathematical model.



                                          The direction of the vector describes direction of planar rotation (right-hand rule) while the magnitude of the vector describes the magnitude of momentum. To speak of angular momentum, imagine two disks of equal size, one made of styrofoam, the other of granite. It will be much harder to accelerate the stone disk than the styrofoam disk. Similarly, once spinning it will be harder to decelerate the stone disk because it has higher angular momentum (due to more mass).






                                          share|cite|improve this answer









                                          $endgroup$




                                          What actually is the vector of angular momentum?




                                          It's the mathematical representation of physical rotation. That is, rotation can be quantified by a vector quantity. That's it. There's nothing "real" about the "vector of angular momentum." It's kind of like using complex numbers to represent wave phases. It's just a nice way to encode a real-world relationship into a mathematical model.



                                          The direction of the vector describes direction of planar rotation (right-hand rule) while the magnitude of the vector describes the magnitude of momentum. To speak of angular momentum, imagine two disks of equal size, one made of styrofoam, the other of granite. It will be much harder to accelerate the stone disk than the styrofoam disk. Similarly, once spinning it will be harder to decelerate the stone disk because it has higher angular momentum (due to more mass).







                                          share|cite|improve this answer












                                          share|cite|improve this answer



                                          share|cite|improve this answer










                                          answered Apr 30 at 21:24









                                          JamesJames

                                          1,57268




                                          1,57268





















                                              0












                                              $begingroup$

                                              You are encountering here an example of the subtle but significant difference between the physical quantity and its mathematical representation. The physical quantity is a measure of the degree to which mass is moving around some axis. There are many ways one could try to quantity that idea. The way that is adopted in physics is to define angular momentum so that it will be a conserved quantity in an isolated system. That is, while the internal parts of the system may move and push each in complicated ways, the total angular momentum of the system will not be affected by all such internal goings-on. This how we arrive at the definition
                                              $$
                                              bf L = bf r times bf v
                                              $$

                                              for the angular momentum of a particle about the origin. I am omitting the proof that this is indeed a nice conserved quantity, when added up over all the parts of an isolated system. For a rigidly rotating object, the resulting angular momentum vector is along the direction you know (i.e. along the axis of rotation, and pointing in the right-handed sense).



                                              What happens next is the mathematics says: "ok; you now have a useful quantity. And according to the way we defined it, it turns out to be a vector which points in the direction at right angles to both $bf r$ and $bf p$. If that direction seems puzzling to you, don't worry---it will become familiar as you see it working correctly in lots of examples. It does not mean anything is moving along the direction of $bf L$. Rather, the direction of $bf L$ indicates the direction of an axis about which some rotation or orbital motion is happening."



                                              That's the end of the answer to your question, but since you mention the example of a bicycle I'll say a little about that too.



                                              In the case of a bicycle the angular momentum associated with rotation of the wheels does not contribute much to the stability. What is much more important is the angle between the front fork and the axle, which determines how the weight of the bike tends to align the front wheel. However, just for interest, let's see what happens to the angular momentum of the wheels when you ride a bike.



                                              As you sit on the bike rolling forward, the angular momentum vector of either wheel is pointing to the left. (Use your right hand to check this). If we wish to turn the bike towards the right, we require, among other things, a torque to be applied to the front wheel so as to rotate its angular momentum vector. To make the correct change for a right turn the torque vector should be directed forwards, because we want the change $Delta bf L$ to be in the forwards direction, in order to swing $bf L$ around in the correct sense. One way to provide such a torque vector is to tip the wheel over, and the interesting point is that you should tip the wheel in the very sense that in any case you require in order to turn the bike, which is to say tipping into the corner, towards the right as you take a right hand bend (and towards the left as you take a left hand bend). (This tipping of the bike also ensures that the combination of friction and normal reaction from the road surface does not flip the bike over as you take the turn, and in fact this is its main role.)






                                              share|cite|improve this answer









                                              $endgroup$

















                                                0












                                                $begingroup$

                                                You are encountering here an example of the subtle but significant difference between the physical quantity and its mathematical representation. The physical quantity is a measure of the degree to which mass is moving around some axis. There are many ways one could try to quantity that idea. The way that is adopted in physics is to define angular momentum so that it will be a conserved quantity in an isolated system. That is, while the internal parts of the system may move and push each in complicated ways, the total angular momentum of the system will not be affected by all such internal goings-on. This how we arrive at the definition
                                                $$
                                                bf L = bf r times bf v
                                                $$

                                                for the angular momentum of a particle about the origin. I am omitting the proof that this is indeed a nice conserved quantity, when added up over all the parts of an isolated system. For a rigidly rotating object, the resulting angular momentum vector is along the direction you know (i.e. along the axis of rotation, and pointing in the right-handed sense).



                                                What happens next is the mathematics says: "ok; you now have a useful quantity. And according to the way we defined it, it turns out to be a vector which points in the direction at right angles to both $bf r$ and $bf p$. If that direction seems puzzling to you, don't worry---it will become familiar as you see it working correctly in lots of examples. It does not mean anything is moving along the direction of $bf L$. Rather, the direction of $bf L$ indicates the direction of an axis about which some rotation or orbital motion is happening."



                                                That's the end of the answer to your question, but since you mention the example of a bicycle I'll say a little about that too.



                                                In the case of a bicycle the angular momentum associated with rotation of the wheels does not contribute much to the stability. What is much more important is the angle between the front fork and the axle, which determines how the weight of the bike tends to align the front wheel. However, just for interest, let's see what happens to the angular momentum of the wheels when you ride a bike.



                                                As you sit on the bike rolling forward, the angular momentum vector of either wheel is pointing to the left. (Use your right hand to check this). If we wish to turn the bike towards the right, we require, among other things, a torque to be applied to the front wheel so as to rotate its angular momentum vector. To make the correct change for a right turn the torque vector should be directed forwards, because we want the change $Delta bf L$ to be in the forwards direction, in order to swing $bf L$ around in the correct sense. One way to provide such a torque vector is to tip the wheel over, and the interesting point is that you should tip the wheel in the very sense that in any case you require in order to turn the bike, which is to say tipping into the corner, towards the right as you take a right hand bend (and towards the left as you take a left hand bend). (This tipping of the bike also ensures that the combination of friction and normal reaction from the road surface does not flip the bike over as you take the turn, and in fact this is its main role.)






                                                share|cite|improve this answer









                                                $endgroup$















                                                  0












                                                  0








                                                  0





                                                  $begingroup$

                                                  You are encountering here an example of the subtle but significant difference between the physical quantity and its mathematical representation. The physical quantity is a measure of the degree to which mass is moving around some axis. There are many ways one could try to quantity that idea. The way that is adopted in physics is to define angular momentum so that it will be a conserved quantity in an isolated system. That is, while the internal parts of the system may move and push each in complicated ways, the total angular momentum of the system will not be affected by all such internal goings-on. This how we arrive at the definition
                                                  $$
                                                  bf L = bf r times bf v
                                                  $$

                                                  for the angular momentum of a particle about the origin. I am omitting the proof that this is indeed a nice conserved quantity, when added up over all the parts of an isolated system. For a rigidly rotating object, the resulting angular momentum vector is along the direction you know (i.e. along the axis of rotation, and pointing in the right-handed sense).



                                                  What happens next is the mathematics says: "ok; you now have a useful quantity. And according to the way we defined it, it turns out to be a vector which points in the direction at right angles to both $bf r$ and $bf p$. If that direction seems puzzling to you, don't worry---it will become familiar as you see it working correctly in lots of examples. It does not mean anything is moving along the direction of $bf L$. Rather, the direction of $bf L$ indicates the direction of an axis about which some rotation or orbital motion is happening."



                                                  That's the end of the answer to your question, but since you mention the example of a bicycle I'll say a little about that too.



                                                  In the case of a bicycle the angular momentum associated with rotation of the wheels does not contribute much to the stability. What is much more important is the angle between the front fork and the axle, which determines how the weight of the bike tends to align the front wheel. However, just for interest, let's see what happens to the angular momentum of the wheels when you ride a bike.



                                                  As you sit on the bike rolling forward, the angular momentum vector of either wheel is pointing to the left. (Use your right hand to check this). If we wish to turn the bike towards the right, we require, among other things, a torque to be applied to the front wheel so as to rotate its angular momentum vector. To make the correct change for a right turn the torque vector should be directed forwards, because we want the change $Delta bf L$ to be in the forwards direction, in order to swing $bf L$ around in the correct sense. One way to provide such a torque vector is to tip the wheel over, and the interesting point is that you should tip the wheel in the very sense that in any case you require in order to turn the bike, which is to say tipping into the corner, towards the right as you take a right hand bend (and towards the left as you take a left hand bend). (This tipping of the bike also ensures that the combination of friction and normal reaction from the road surface does not flip the bike over as you take the turn, and in fact this is its main role.)






                                                  share|cite|improve this answer









                                                  $endgroup$



                                                  You are encountering here an example of the subtle but significant difference between the physical quantity and its mathematical representation. The physical quantity is a measure of the degree to which mass is moving around some axis. There are many ways one could try to quantity that idea. The way that is adopted in physics is to define angular momentum so that it will be a conserved quantity in an isolated system. That is, while the internal parts of the system may move and push each in complicated ways, the total angular momentum of the system will not be affected by all such internal goings-on. This how we arrive at the definition
                                                  $$
                                                  bf L = bf r times bf v
                                                  $$

                                                  for the angular momentum of a particle about the origin. I am omitting the proof that this is indeed a nice conserved quantity, when added up over all the parts of an isolated system. For a rigidly rotating object, the resulting angular momentum vector is along the direction you know (i.e. along the axis of rotation, and pointing in the right-handed sense).



                                                  What happens next is the mathematics says: "ok; you now have a useful quantity. And according to the way we defined it, it turns out to be a vector which points in the direction at right angles to both $bf r$ and $bf p$. If that direction seems puzzling to you, don't worry---it will become familiar as you see it working correctly in lots of examples. It does not mean anything is moving along the direction of $bf L$. Rather, the direction of $bf L$ indicates the direction of an axis about which some rotation or orbital motion is happening."



                                                  That's the end of the answer to your question, but since you mention the example of a bicycle I'll say a little about that too.



                                                  In the case of a bicycle the angular momentum associated with rotation of the wheels does not contribute much to the stability. What is much more important is the angle between the front fork and the axle, which determines how the weight of the bike tends to align the front wheel. However, just for interest, let's see what happens to the angular momentum of the wheels when you ride a bike.



                                                  As you sit on the bike rolling forward, the angular momentum vector of either wheel is pointing to the left. (Use your right hand to check this). If we wish to turn the bike towards the right, we require, among other things, a torque to be applied to the front wheel so as to rotate its angular momentum vector. To make the correct change for a right turn the torque vector should be directed forwards, because we want the change $Delta bf L$ to be in the forwards direction, in order to swing $bf L$ around in the correct sense. One way to provide such a torque vector is to tip the wheel over, and the interesting point is that you should tip the wheel in the very sense that in any case you require in order to turn the bike, which is to say tipping into the corner, towards the right as you take a right hand bend (and towards the left as you take a left hand bend). (This tipping of the bike also ensures that the combination of friction and normal reaction from the road surface does not flip the bike over as you take the turn, and in fact this is its main role.)







                                                  share|cite|improve this answer












                                                  share|cite|improve this answer



                                                  share|cite|improve this answer










                                                  answered Apr 30 at 21:36









                                                  Andrew SteaneAndrew Steane

                                                  6,1251737




                                                  6,1251737





















                                                      0












                                                      $begingroup$

                                                      Whenever you see the cross product, know you’re dealing with a second rank antisymmetric tensor in disguise. Second rank tensors acting on 3D vectors can be represented by $3times3$ matrices, and if that matrix is required to be antisymmetric, that puts six constraints on it, leaving three degrees of freedom to define it. Thus the action of the antisymmetric tensor



                                                      $$A=beginpmatrix
                                                      0 & a_1 & a_2 \
                                                      -a_1 & 0 & a_3 \
                                                      -a_2 & -a_3 & 0
                                                      endpmatrix$$



                                                      on a vector $mathbfv,$ that is, $Amathbfv,$ can be conveniently denoted as $mathbfa times mathbfv,$ where $mathbfa = (-a_3, a_2, -a_1)^mathrmT.$ Let’s check:



                                                      $$beginpmatrixa_x \ a_y \ a_zendpmatrix times
                                                      beginpmatrixv_x \ v_y \ v_zendpmatrix =
                                                      beginpmatrix
                                                      0 & -a_z & a_y \
                                                      a_z & 0 & -a_x \
                                                      -a_y & a_x & 0
                                                      endpmatrix
                                                      beginpmatrixv_x \ v_y \ v_zendpmatrix =
                                                      beginpmatrix
                                                      -a_z v_y+a_yv_z \
                                                      phantom-a_zv_x - a_xv_z \
                                                      -a_yv_x+a_xv_y
                                                      endpmatrix
                                                      $$

                                                      which matches the definition of the cross product in terms of the Cartesian coordinates.



                                                      So it’s just a notational trick based on the fact that in 3D space (and in no other number of dimensions), rank 2 antisymmetric tensors have exactly as many degrees of freedom as vectors and can be put into correspondence. These pseudo-vectors end up pointing in directions which don’t necessarily mean anything is moving in that direction, and their magnitudes might be devoid of physical meaning as well.



                                                      A similar approach can be used with other kinds of tensors, possibly requiring more than one vector to match the amount of information contained in the tensor. This is one way to define the $mathbfE$ and $mathbfB$ vectors in electromagnetism theory, while tensor forms of the laws combine those quantities in a single entity.






                                                      share|cite|improve this answer









                                                      $endgroup$

















                                                        0












                                                        $begingroup$

                                                        Whenever you see the cross product, know you’re dealing with a second rank antisymmetric tensor in disguise. Second rank tensors acting on 3D vectors can be represented by $3times3$ matrices, and if that matrix is required to be antisymmetric, that puts six constraints on it, leaving three degrees of freedom to define it. Thus the action of the antisymmetric tensor



                                                        $$A=beginpmatrix
                                                        0 & a_1 & a_2 \
                                                        -a_1 & 0 & a_3 \
                                                        -a_2 & -a_3 & 0
                                                        endpmatrix$$



                                                        on a vector $mathbfv,$ that is, $Amathbfv,$ can be conveniently denoted as $mathbfa times mathbfv,$ where $mathbfa = (-a_3, a_2, -a_1)^mathrmT.$ Let’s check:



                                                        $$beginpmatrixa_x \ a_y \ a_zendpmatrix times
                                                        beginpmatrixv_x \ v_y \ v_zendpmatrix =
                                                        beginpmatrix
                                                        0 & -a_z & a_y \
                                                        a_z & 0 & -a_x \
                                                        -a_y & a_x & 0
                                                        endpmatrix
                                                        beginpmatrixv_x \ v_y \ v_zendpmatrix =
                                                        beginpmatrix
                                                        -a_z v_y+a_yv_z \
                                                        phantom-a_zv_x - a_xv_z \
                                                        -a_yv_x+a_xv_y
                                                        endpmatrix
                                                        $$

                                                        which matches the definition of the cross product in terms of the Cartesian coordinates.



                                                        So it’s just a notational trick based on the fact that in 3D space (and in no other number of dimensions), rank 2 antisymmetric tensors have exactly as many degrees of freedom as vectors and can be put into correspondence. These pseudo-vectors end up pointing in directions which don’t necessarily mean anything is moving in that direction, and their magnitudes might be devoid of physical meaning as well.



                                                        A similar approach can be used with other kinds of tensors, possibly requiring more than one vector to match the amount of information contained in the tensor. This is one way to define the $mathbfE$ and $mathbfB$ vectors in electromagnetism theory, while tensor forms of the laws combine those quantities in a single entity.






                                                        share|cite|improve this answer









                                                        $endgroup$















                                                          0












                                                          0








                                                          0





                                                          $begingroup$

                                                          Whenever you see the cross product, know you’re dealing with a second rank antisymmetric tensor in disguise. Second rank tensors acting on 3D vectors can be represented by $3times3$ matrices, and if that matrix is required to be antisymmetric, that puts six constraints on it, leaving three degrees of freedom to define it. Thus the action of the antisymmetric tensor



                                                          $$A=beginpmatrix
                                                          0 & a_1 & a_2 \
                                                          -a_1 & 0 & a_3 \
                                                          -a_2 & -a_3 & 0
                                                          endpmatrix$$



                                                          on a vector $mathbfv,$ that is, $Amathbfv,$ can be conveniently denoted as $mathbfa times mathbfv,$ where $mathbfa = (-a_3, a_2, -a_1)^mathrmT.$ Let’s check:



                                                          $$beginpmatrixa_x \ a_y \ a_zendpmatrix times
                                                          beginpmatrixv_x \ v_y \ v_zendpmatrix =
                                                          beginpmatrix
                                                          0 & -a_z & a_y \
                                                          a_z & 0 & -a_x \
                                                          -a_y & a_x & 0
                                                          endpmatrix
                                                          beginpmatrixv_x \ v_y \ v_zendpmatrix =
                                                          beginpmatrix
                                                          -a_z v_y+a_yv_z \
                                                          phantom-a_zv_x - a_xv_z \
                                                          -a_yv_x+a_xv_y
                                                          endpmatrix
                                                          $$

                                                          which matches the definition of the cross product in terms of the Cartesian coordinates.



                                                          So it’s just a notational trick based on the fact that in 3D space (and in no other number of dimensions), rank 2 antisymmetric tensors have exactly as many degrees of freedom as vectors and can be put into correspondence. These pseudo-vectors end up pointing in directions which don’t necessarily mean anything is moving in that direction, and their magnitudes might be devoid of physical meaning as well.



                                                          A similar approach can be used with other kinds of tensors, possibly requiring more than one vector to match the amount of information contained in the tensor. This is one way to define the $mathbfE$ and $mathbfB$ vectors in electromagnetism theory, while tensor forms of the laws combine those quantities in a single entity.






                                                          share|cite|improve this answer









                                                          $endgroup$



                                                          Whenever you see the cross product, know you’re dealing with a second rank antisymmetric tensor in disguise. Second rank tensors acting on 3D vectors can be represented by $3times3$ matrices, and if that matrix is required to be antisymmetric, that puts six constraints on it, leaving three degrees of freedom to define it. Thus the action of the antisymmetric tensor



                                                          $$A=beginpmatrix
                                                          0 & a_1 & a_2 \
                                                          -a_1 & 0 & a_3 \
                                                          -a_2 & -a_3 & 0
                                                          endpmatrix$$



                                                          on a vector $mathbfv,$ that is, $Amathbfv,$ can be conveniently denoted as $mathbfa times mathbfv,$ where $mathbfa = (-a_3, a_2, -a_1)^mathrmT.$ Let’s check:



                                                          $$beginpmatrixa_x \ a_y \ a_zendpmatrix times
                                                          beginpmatrixv_x \ v_y \ v_zendpmatrix =
                                                          beginpmatrix
                                                          0 & -a_z & a_y \
                                                          a_z & 0 & -a_x \
                                                          -a_y & a_x & 0
                                                          endpmatrix
                                                          beginpmatrixv_x \ v_y \ v_zendpmatrix =
                                                          beginpmatrix
                                                          -a_z v_y+a_yv_z \
                                                          phantom-a_zv_x - a_xv_z \
                                                          -a_yv_x+a_xv_y
                                                          endpmatrix
                                                          $$

                                                          which matches the definition of the cross product in terms of the Cartesian coordinates.



                                                          So it’s just a notational trick based on the fact that in 3D space (and in no other number of dimensions), rank 2 antisymmetric tensors have exactly as many degrees of freedom as vectors and can be put into correspondence. These pseudo-vectors end up pointing in directions which don’t necessarily mean anything is moving in that direction, and their magnitudes might be devoid of physical meaning as well.



                                                          A similar approach can be used with other kinds of tensors, possibly requiring more than one vector to match the amount of information contained in the tensor. This is one way to define the $mathbfE$ and $mathbfB$ vectors in electromagnetism theory, while tensor forms of the laws combine those quantities in a single entity.







                                                          share|cite|improve this answer












                                                          share|cite|improve this answer



                                                          share|cite|improve this answer










                                                          answered Apr 30 at 23:03









                                                          Roman OdaiskyRoman Odaisky

                                                          1211




                                                          1211



























                                                              draft saved

                                                              draft discarded
















































                                                              Thanks for contributing an answer to Physics Stack Exchange!


                                                              • Please be sure to answer the question. Provide details and share your research!

                                                              But avoid


                                                              • Asking for help, clarification, or responding to other answers.

                                                              • Making statements based on opinion; back them up with references or personal experience.

                                                              Use MathJax to format equations. MathJax reference.


                                                              To learn more, see our tips on writing great answers.




                                                              draft saved


                                                              draft discarded














                                                              StackExchange.ready(
                                                              function ()
                                                              StackExchange.openid.initPostLogin('.new-post-login', 'https%3a%2f%2fphysics.stackexchange.com%2fquestions%2f476952%2fwhat-actually-is-the-vector-of-angular-momentum%23new-answer', 'question_page');

                                                              );

                                                              Post as a guest















                                                              Required, but never shown





















































                                                              Required, but never shown














                                                              Required, but never shown












                                                              Required, but never shown







                                                              Required, but never shown

































                                                              Required, but never shown














                                                              Required, but never shown












                                                              Required, but never shown







                                                              Required, but never shown







                                                              Popular posts from this blog

                                                              Wikipedia:Vital articles Мазмуну Biography - Өмүр баян Philosophy and psychology - Философия жана психология Religion - Дин Social sciences - Коомдук илимдер Language and literature - Тил жана адабият Science - Илим Technology - Технология Arts and recreation - Искусство жана эс алуу History and geography - Тарых жана география Навигация менюсу

                                                              Bruxelas-Capital Índice Historia | Composición | Situación lingüística | Clima | Cidades irmandadas | Notas | Véxase tamén | Menú de navegacióneO uso das linguas en Bruxelas e a situación do neerlandés"Rexión de Bruxelas Capital"o orixinalSitio da rexiónPáxina de Bruselas no sitio da Oficina de Promoción Turística de Valonia e BruxelasMapa Interactivo da Rexión de Bruxelas-CapitaleeWorldCat332144929079854441105155190212ID28008674080552-90000 0001 0666 3698n94104302ID540940339365017018237

                                                              What should I write in an apology letter, since I have decided not to join a company after accepting an offer letterShould I keep looking after accepting a job offer?What should I do when I've been verbally told I would get an offer letter, but still haven't gotten one after 4 weeks?Do I accept an offer from a company that I am not likely to join?New job hasn't confirmed starting date and I want to give current employer as much notice as possibleHow should I address my manager in my resignation letter?HR delayed background verification, now jobless as resignedNo email communication after accepting a formal written offer. How should I phrase the call?What should I do if after receiving a verbal offer letter I am informed that my written job offer is put on hold due to some internal issues?Should I inform the current employer that I am about to resign within 1-2 weeks since I have signed the offer letter and waiting for visa?What company will do, if I send their offer letter to another company